بالکان

yasamin.m

New Member
ارسال ها
43
لایک ها
4
امتیاز
0
#1
سال نو مبارک:3:
فرض کنید nعددی صحیح مثبت و

نشان دهید
 

MBGO

New Member
ارسال ها
247
لایک ها
104
امتیاز
0
#2
پاسخ : بالکان

با سلام
سال نو مبارک*_^

این سوال رو یه سری ها شک دارند که اصلا درست هست یا نه اما اثبات های مختلفی برای درست بودنش (و همین طور نبودنش) وجود داره. در هر حال من حل کاملی ازش ندیدم و (میگن) خود بالکان هم حل درست و حسابی براش نداشت. در هر حال تو این لینک یه چیزایی در موردش گفته شده.

http://www.artofproblemsolving.com/Forum/viewtopic.php?highlight=1989&t=300

(+ این مساله تو کجای بالکان بوده؟)

با تشکر.
 

mimilad

New Member
ارسال ها
298
لایک ها
40
امتیاز
0
#3
پاسخ : بالکان

کجای سوال مبهمه خیلی راحته که فقط یه همنهشتی و استفاده از مرتبه اس .
 

MBGO

New Member
ارسال ها
247
لایک ها
104
امتیاز
0
#4
پاسخ : بالکان

اگه میتونی راه حل رو بذاری ممنون میشم چون راه حل کاملی ازش ندیدم.

با تشکر.
 

mimilad

New Member
ارسال ها
298
لایک ها
40
امتیاز
0
#5
پاسخ : بالکان

مرتبه به ÷یمانه 17 و 9 و13 رو بگیر و ثابت کن n^n^n-n^n توسط 144 عاد میشه (فک کنم همین کافی باشه ):3:
 
ارسال ها
317
لایک ها
151
امتیاز
0
#6
پاسخ : بالکان

با سلام
سال نو مبارک*_^

این سوال رو یه سری ها شک دارند که اصلا درست هست یا نه اما اثبات های مختلفی برای درست بودنش (و همین طور نبودنش) وجود داره. در هر حال من حل کاملی ازش ندیدم و (میگن) خود بالکان هم حل درست و حسابی براش نداشت. در هر حال تو این لینک یه چیزایی در موردش گفته شده.

AoPS Forum - balkan89unused • Art of Problem Solving

(+ این مساله تو کجای بالکان بوده؟)

با تشکر.

:13::13: چی میگی ماهان این که بدیهیه خیلی راحت v[SUB]p[/SUB] شو میتونی بگیری
 
ارسال ها
317
لایک ها
151
امتیاز
0
#7
پاسخ : بالکان

خوب اگه جوب زدم بگین


البته اینم واسه لما و اثباتاش Lifting The Exponent Lemma

یکی به من pm داد که باید پایه ویه پی رو ثابت کنی که P میشمارتش اما اگه (p,توان مورد نظر)=1 باشه چی؟!!!!!!!!اون موقع دیگه نمیخواد
 
آخرین ویرایش توسط مدیر
ارسال ها
317
لایک ها
151
امتیاز
0
#9
آخرین ویرایش توسط مدیر
ارسال ها
317
لایک ها
151
امتیاز
0
#10
پاسخ : بالکان

تازه کی گفته
به ازای
برقرار نیس
 

mimilad

New Member
ارسال ها
298
لایک ها
40
امتیاز
0
#11
پاسخ : بالکان

اعداد به شکل 4k+2 مستقیما تو قسمت سومی که توسط 16 عاد میشن (البته به جز دو که تو دامنه ما نیست )
 
آخرین ویرایش توسط مدیر

mimilad

New Member
ارسال ها
298
لایک ها
40
امتیاز
0
#12
پاسخ : بالکان

به حالا هرکی :

خوب مرتبه ی a به ÷یمانه ی p تمام مضارب p-1 رو عاد میکنه . (اون که اول گفتی هم دلیلش این بود )
تنها چیز دیکه ای که استفاده شد استفاده از مرتبه ی اعداد فرد به ÷یمانه ی 16 بود
 
آخرین ویرایش توسط مدیر
ارسال ها
317
لایک ها
151
امتیاز
0
#13
پاسخ : بالکان

آره درسته قشنگ بود
آفرین
 
بالا